LSAT and Law School Admissions Forum

Get expert LSAT preparation and law school admissions advice from PowerScore Test Preparation.

User avatar
 Dave Killoran
PowerScore Staff
  • PowerScore Staff
  • Posts: 5849
  • Joined: Mar 25, 2011
|
#26433
Complete Question Explanation
(The complete setup for this game can be found here: lsat/viewtopic.php?t=11009)

The correct answer choice is (B)

If P received more votes than J, then six of the variables in the following sequence: L-P-J-O-K-M in order. N must then float somewhere after P but before M. Thus, L and P must always be first and second, and M must always be last. Therefore, if P received more votes than J, then three would be the maximum possible number of names whose ranks could be determined. It follows that answer choice (B) is correct.

Note that the third position often troubles students, with the thought that perhaps J must be third. But N could also be third since it must only have fewer votes than P. Here's every possible order of the variables in this question, with the variables that are fixed in one spot in bold:

  • N is 3: ..... L-P-N-J-O-K-M

    N is 4: ..... L-P-J-N-O-K-M

    N is 5: ..... L-P-J-O-N-K-M

    N is 6: ..... L-P-J-O-K-N-M
 jennamaree18
  • Posts: 1
  • Joined: Jul 21, 2017
|
#37513
Why does P have to be second?
User avatar
 Dave Killoran
PowerScore Staff
  • PowerScore Staff
  • Posts: 5849
  • Joined: Mar 25, 2011
|
#37527
Hi Jenna,

Thanks for the question! In the diagram, we know that P receives more votes than O and N. O already receives more votes that K and M ( O :longline: K :longline: M), so right away P is a relatively high-positioned vote getter. Now, We also know that L receives more votes than P, so P isn't first. Thus, P is actually quite limited in where it can be just based on the original rules.

Now, when the question stem stipulates that "Ping received more votes than Jazz," We know that P receives more votes than J, O, K, M, and N—that's 5 soft drinks, meaning P is first or second! But, since L receives more votes than P, so P isn't first and thus P must be second in this question.

Please let me know if that helps. Thanks!
User avatar
 Dave Killoran
PowerScore Staff
  • PowerScore Staff
  • Posts: 5849
  • Joined: Mar 25, 2011
|
#64297
Catherine96 wrote:Hello!

I was wondering why we couldn't determine J to be in the 3rd spot. Is it because N could be in the third spot (followed by J then O)?

Thanks,

Catherine
Hi Catherine,

Yes, that is exactly right. Well done!

From the question stem, we can determine that we have L-P-J-O-K-M in order, with N floating around somewhere after P but before M. So, when we get to that third spot, we can have either J or N.

Just for fun, here's every possible order of the variables in this question, with the variables that are fixed in one spot in bold:

  • N is 3: ..... L-P-N-J-O-K-M

    N is 4: ..... L-P-J-N-O-K-M

    N is 5: ..... L-P-J-O-N-K-M

    N is 6: ..... L-P-J-O-K-N-M
I've also amended the explanation at the top to show this. Please let me know if that helps. Thanks!
 kupwarriors9
  • Posts: 73
  • Joined: Jul 01, 2021
|
#88758
Is there a more efficient way of doing this, instead of writing it out each time? Thanks!
Dave Killoran wrote: Sat Jun 11, 2016 11:00 am Complete Question Explanation
(The complete setup for this game can be found here: lsat/viewtopic.php?t=11009)

The correct answer choice is (B)

If P received more votes than J, then six of the variables in the following sequence: L-P-J-O-K-M in order. N must then float somewhere after P but before M. Thus, L and P must always be first and second, and M must always be last. Therefore, if P received more votes than J, then three would be the maximum possible number of names whose ranks could be determined. It follows that answer choice (B) is correct.

Note that the third position often troubles students, with the thought that perhaps J must be third. But N could also be third since it must only have fewer votes than P. Here's every possible order of the variables in this question, with the variables that are fixed in one spot in bold:

  • N is 3: ..... L-P-N-J-O-K-M

    N is 4: ..... L-P-J-N-O-K-M

    N is 5: ..... L-P-J-O-N-K-M

    N is 6: ..... L-P-J-O-K-N-M
User avatar
 Dave Killoran
PowerScore Staff
  • PowerScore Staff
  • Posts: 5849
  • Joined: Mar 25, 2011
|
#88771
Yes, the first paragraph of my explanation outlines how I would typically solve this. The listing of solutions was done in response to a specific student question afterward, where I prefaced my reply with "just for fun." So, that's just showing what can occur as a matter of post-explanation :-D
 aamcgowa
  • Posts: 10
  • Joined: Jul 17, 2021
|
#92716
Hello,
Why must N be before M?
 Rachael Wilkenfeld
PowerScore Staff
  • PowerScore Staff
  • Posts: 1358
  • Joined: Dec 15, 2011
|
#92728
Hi aamcgowa,

Remember rule four---N can not go last. Therefore, N has to be before M, or it would be last. This is one of the global rules about the game, not just local to this question.

Hope that helps!
 zworkman
  • Posts: 1
  • Joined: Jun 06, 2023
|
#102055
For question 12, I do not understand how the answer would not be more than three. I thought it would be 5 After doing the diagram. I know for sure that L and P have their spots. From my diagram it's:
1. L
2. P
3. J or N
4. J or N
5. O
6. K
7. M
User avatar
 Dave Killoran
PowerScore Staff
  • PowerScore Staff
  • Posts: 5849
  • Joined: Mar 25, 2011
|
#102073
Hi Z,

Above, I listed out ALL of the possible solutions to this question, and they are different from what you have. Specifically, you've locked in O and K as 5th and 6th, and while that is possible there are other variations as well (namely N-K and K-N). Take a look at that point specifically because that's where you got tripped up here.

Thanks!

Get the most out of your LSAT Prep Plus subscription.

Analyze and track your performance with our Testing and Analytics Package.